Musculoskeletal Path Flashcards

1
Q

A 78 year old woman comes to her physician due to new onset continuous back pain. She is diagnosed with a vertebral fracture and the following is seen on her x-ray. What is her diagnosis- what caused this injury?

A

Osteoporosis:
Primary= more common
1. Type 1= post-menopausal; absolute increase in osteoclast activity due to estrogen withdrawal
2. Type 2= > 70 years, attenuated osteoblast function (no increase in osteoclast activity)

Secondary osteoporosis: Associated with other conditions:

  • Endocrine (Cushing’s, hyperthyroidism, hypogonadism, iatrogenic steroids–> inhibited osteoblasts)
  • Hematologic malignancies (Multiple myeloma–> activates osteoclasts
  • Malabsorption (decreased Ca, Ph, Vit D)
  • Alcoholism–> directly inhibits osteoblasts
How well did you know this?
1
Not at all
2
3
4
5
Perfectly
2
Q

A 58 year old man comes to his physician with increasing pain in his knees that worsens with activity. Suspecting arthritis, the physician orders an x-ray that reveals unusually dense bones. Additionally the man has had anemia refractory to iron supplementation. Based on these two clinical features, what is his possible diagnosis and treatment? What caused this pathology?

A

Osteopetrosis:
Marble bone disease: Albers-Schonberg disease
- Group of rare, heritable disorders; increased skeletal mass due to abnormally dense bone

  • *Short, block-like radiodense bones on x-ray/CT**
  • Radiopaque, 2-3 times normal weight
  • WEAK bone due to disorganized structure (can’t remodel with stress)
  • Calcified cartilage= weak, friable
  • *Anemia due to replacement of marrow by sheets of osteoclasts, marrow fibrosis**
  • Extramedullary hematopoeisis
  • Pancytopenia

Blindness, deafness (foramen entraps nerves)

Treatment: bone marrow transplant, human IFN-gamma

  • *Types:
    1. **Autosomal recessive= severe, fatal to infants
  • Marked anemia, cranial nerve entrapment, hydrocephalus, infection
  1. Autosomal dominant= mild anemia/ asymptomatic

Pathogenesis:

  • Failed osteoclastic bone resorption
  • Mutation in osteoclast formation/function genes
  • Defect in acidification of bone
  • Mutations: TCIRG1 (AD- proton pump); CLCN7 (AR- Chloride channel), Carbonic anhydrase II (AR); transciption factors, cytokines causing osteoclast differentiation
  • Primary spongiosum remains–> cartilage cores, thickened cortex, lack of funnelization of metaphysis
How well did you know this?
1
Not at all
2
3
4
5
Perfectly
3
Q

A 35 year old man with a 15 year history of Ulcerative Colitis fractures his arm after falling in a soccer game. The x-ray of his arm reveals decreased bone density. What is his diagnosis and what caused this?

A

Osteomalacia:
Inadequate bone mineralization of newly formed bone, cartilage matrix
- Adults and kids–> osteopenia

Causes:

  • *- abnormal Vit D metabolism
  • Intestinal malabsorption**
  • Renal disorders
  • Phosphate deficiency
  • mineralization defects

Path:

  • pseudofractures
  • Exaggerated osteoid seams (osteoclast activity can’t occur, delayed calcification)
How well did you know this?
1
Not at all
2
3
4
5
Perfectly
4
Q

A 65 year old woman comes to her physician complaining of pain in her head and light headedness as well as arthritis. Her physician is concerned she may have migraines but on DXA scan she has extremely low bone density. X-ray reveals abnormal bone formation. Based on her clinical symptoms and her x-ray, what is her diagnosis and what other diseases could she develop?

A

Paget’s disease:
Pain (focal feature)
- Skull involvement (frontal/parietal)
- Pagetic steal–> light headedness (calvareal bone is stealing blood supply)
- Fractures/ arthritis
- High output cardiac failure (significant part of skeleton involved–> increased cardiac demands)
- Sarcomatous change, osteosarcoma in older patients (< 1%)

Chronic condition: bone enlargement and lytic lesions (disordered remodeling)

  • uncoupling osteoblast/osteoclast activity
  • See clast only activity, or blastic activity with patchy clast activity–> mosaic bone
  • Seen in M & F > 60 years (3-4% in certain populations)
  • Solitary or multiple sites

Pathogenesis: hereditary AD (chr 18)

  • Sequestosome 1–> p62 protein in RANK pathway (more aggressive form)
  • Viral nuclear inclusions similar to SSE (subacute sclerosing encephalitis)- giant cell tumor of bone

Path:

  • “Hot” osteoclastic stage= lytic
  • Mixed stage
  • “cold” burnout stage: little activity

Histo:

  • _Prominent cement lines (“mosaic”)
  • Osteoclast= pathologic cell; “killer clasts” with 50+ nuclei (vs normal 8)_
  • Can see sarcomatous change in < 1% (adult type of osteosarcoma)
How well did you know this?
1
Not at all
2
3
4
5
Perfectly
5
Q

A 68 year old woman with multiple recent fractures is at the orthopedist’s office for further evaluation of her condition. On x-ray there is a lesion in her bone that looks suspiciously different from her previous diagnosis of osteoporosis. A bone biopsy reveals atypia and hypercellularity. What is her diagnosis and what caused this?

A

Osteosarcoma due to Paget’s disease: sarcoma formation:
Pain (focal feature)
- Skull involvement (frontal/parietal)
- Pagetic steal–> light headedness (calvareal bone is stealing blood supply)
- Fractures/ arthritis
- High output cardiac failure (significant part of skeleton involved–> increased cardiac demands)
- Sarcomatous change, osteosarcoma in older patients (< 1%)

Chronic condition: bone enlargement and lytic lesions (disordered remodeling)

  • uncoupling osteoblast/osteoclast activity
  • See clast only activity, or blastic activity with patchy clast activity–> mosaic bone
  • Seen in M & F > 60 years (3-4% in certain populations)
  • Solitary or multiple sites

Pathogenesis: hereditary AD (chr 18)

How well did you know this?
1
Not at all
2
3
4
5
Perfectly
6
Q

A 35 year old Asian woman presents with a fracture in her arm with no previous trauma. An x-ray reveals a lobulated, lytic lesion. Bone biopsy reveals the following cell type. What is her diagnosis and treatment?

A

Giant cell tumor:
Locally aggressive
-20s-40s F > M, Asians
- Site: epiphysis, metaphysis of long bones

Clinical: pain, path fractures

X-ray: lytic, multilobulated “soap bubble”

Gross: Blood-filled sponge

Histo: mononuclear spindle-shaped cells and mononucleated giant cell tumors

Treatment: curettage, recur (50%), 10-15% lung mets

How well did you know this?
1
Not at all
2
3
4
5
Perfectly
7
Q
A

Osteochondroma

Developmental defect (hamartoma) at epiphyseal plate

  • “Ring of Ranvier”
  • Cartilage capped growth growing away from joint

Hereditary multiple osteochondromatosis:
- Extra genes on Chromosomes 8, 11, 19–> chondrocyte proliferation and differentiation

How well did you know this?
1
Not at all
2
3
4
5
Perfectly
8
Q

A 15 year old boy is brought to his family physician due to increasing pain in his hip. An x-ray revels a lytic lesion in the head of his femur. Additionally his alkaline phosphatase levels are elevated on a blood panel. He is sent to a surgeon to have the head of the femur replaced. Based on the specimen and the woven bone with atypical osteoblasts found within, what is his diagnosis?

A

Osteosarcoma

Most common PRIMARY malignant bone-forming tumor (most common bone tumor= metastatic)

  • 2nd decade, M > F
  • Rb gene mutation in 2/3; p53

Sites: metaphysis of femur, tibia > humerus

X-ray: lytic, sunburst pattern, codman’s triangle

Clinical: Pain, increased alk phos

Histo: woven bone (osteoid) with malignant osteoblasts

Variants:

  • Parosteal
  • Periosteal
  • Telangiectatic
How well did you know this?
1
Not at all
2
3
4
5
Perfectly
9
Q

A 15 year old boy presents with pain and swelling in his femur. A bone biopsy revelas the following small round blue cells that stain positive for CD99 and PAS. What is his diagnosis and treatment?

A

Ewing sarcoma:
Uncommon
- 2/3 < 20 years, M > F (2:1)
- Reciprocal translocation t(11;22)(p13;q12), PNET family

Clinical: pain, swelling

X-ray: midshaft/diaphysis of long bones; “onion skin pattern”

Histo: small round blue cell tumor

  • PAS, CD99
  • t(11;22) or t(21;22)

Treatment:

  • Chemo/radiation
  • surgery
  • 60-75% 5-year recurrence rate
How well did you know this?
1
Not at all
2
3
4
5
Perfectly
10
Q

A 15 year old boy presents with pain and swelling in his femur. A bone biopsy revelas the following small round blue cells that stain positive for CD99 and PAS. What is his diagnosis and treatment?

A

Ewing sarcoma:
Uncommon
- 2/3 < 20 years, M > F (2:1)
- Reciprocal translocation t(11;22)(p13;q12), PNET family

Clinical: pain, swelling

X-ray: midshaft/diaphysis of long bones; “onion skin pattern”

Histo: small round blue cell tumor

  • PAS, CD99
  • t(11;22) or t(21;22)

Treatment:

  • Chemo/radiation
  • surgery
  • 60-75% 5-year recurrence rate
How well did you know this?
1
Not at all
2
3
4
5
Perfectly
11
Q

A 42 year old man is treated by his physician for pain in his hip. A CT-scan reveals a lesion in his pelvic bone with calcification and lytic lesions. Histological samples reveal the following. What is his diagnosis?

A

Chondrosarcoma:
2nd most common primary malignant bone tumor
- M > F, 4th-6th decade
- Some from cartilage enchondromas
- Variants classified by location
- Can see molecular trisomy 7, rearrangement of chrom 17
* extracellular sarcoma mixoid type: (9;22)(q31;q12)

Sites:
- Axial, proximal femur, humerus

X-ray: lucent or ring-like; calcified and lytic areas

Histo: grading based on cellularity, degree of atypia

How well did you know this?
1
Not at all
2
3
4
5
Perfectly
12
Q

A 77 year old woman visits an orthopedist due to recurrent, worsening knee pain that now restricts her daily activities of living. Based on the imaging below, what is her diagnosis?

A

Primary osteoarthritis:
Most common form of joint disease
- Slow, progressive
- Seen in articular cartilage of weight bearing joints, fingers
- Older patients (or young following trauma)

  1. Primary osteoarthritis:
    Unknown etiology; several postulated factors:
    - increases with age
    - 85% of patients age 75-79
    - < 45 years= male predominant; > 55 years= females
    - Hereditary factors?
  2. Secondary osteoarthritis:
    Known etiology

Clinical:
See joint space narrowing, subchondral bone thickening, painful joint
- Pain follows activity
- Restriction of motion
- Lab findings unhelpful
- Supportive therapy

Pathology:

  • Seen at DIP, PIP of upper extremity; knees, hips, cervical and lumbar spin
  • Narrowing of joint space on x-ray
  • Increased subcondral bone, bone cysts
  • Osteophyte formation

Histology:

  • Earliest= death of articular cartilage
  • Cracking, fibrillation
  • Reactive bone/cartilage–> fibrocartilage plug
  • Eburnation (loss of articular cartilage- BELOW), subchondral bone cyst (decreased density on x-ray), osteophyte (dense bone formation with cartilage cap, continuous marrow)
How well did you know this?
1
Not at all
2
3
4
5
Perfectly
13
Q

A 77 year old woman visits an orthopedist due to recurrent, worsening knee pain that now restricts her daily activities of living. Based on the gross specimen below, what is her diagnosis?

A

Primary osteoarthritis subchondrial cyst:
Most common form of joint disease
- Slow, progressive
- Seen in articular cartilage of weight bearing joints, fingers
- Older patients (or young following trauma)

  1. Primary osteoarthritis:
    Unknown etiology; several postulated factors:
    - increases with age
    - 85% of patients age 75-79
    - < 45 years= male predominant; > 55 years= females
    - Hereditary factors?
  2. Secondary osteoarthritis:
    Known etiology

Clinical:
See joint space narrowing, subchondral bone thickening, painful joint
- Pain follows activity
- Restriction of motion
- Lab findings unhelpful
- Supportive therapy

Pathology:

  • Seen at DIP, PIP of upper extremity; knees, hips, cervical and lumbar spin
  • Narrowing of joint space on x-ray
  • Increased subcondral bone, bone cysts
  • *- Osteophyte formation**

Histology:

  • Earliest= death of articular cartilage
  • Cracking, fibrillation
  • Reactive bone/cartilage–> fibrocartilage plug
  • Eburnation (loss of articular cartilage), subchondral bone cyst (decreased density on x-ray), osteophyte (dense bone formation with cartilage cap, continuous marrow)
How well did you know this?
1
Not at all
2
3
4
5
Perfectly
14
Q

A 77 year old woman visits an orthopedist due to recurrent, worsening knee pain that now restricts her daily activities of living. Based on the histologic sample below, what is her diagnosis?

A

Primary osteoarthritis: osteophyte:
Most common form of joint disease
- Slow, progressive
- Seen in articular cartilage of weight bearing joints, fingers
- Older patients (or young following trauma)

  1. Primary osteoarthritis:
    Unknown etiology; several postulated factors:
    - increases with age
    - 85% of patients age 75-79
    - < 45 years= male predominant; > 55 years= females
    - Hereditary factors?
  2. Secondary osteoarthritis:
    Known etiology

Clinical:
See joint space narrowing, subchondral bone thickening, painful joint
- Pain follows activity
- Restriction of motion
- Lab findings unhelpful
- Supportive therapy

Pathology:

  • Seen at DIP, PIP of upper extremity; knees, hips, cervical and lumbar spin
  • Narrowing of joint space on x-ray
  • Increased subcondral bone, bone cysts
  • Osteophyte formation
How well did you know this?
1
Not at all
2
3
4
5
Perfectly
15
Q

A 50 year old man with a 20-year history of rheumatoid arthritis comes to the emergency room with a painful swelling in his calf (below). Based on his condition, what could possible be occuring and what is he as risk for?

A

Rheumatoid arthritis of the knee: Ruptured Baker’s cyst
Clinical presentation:
Bilateral, symmetrical
Large and small joints
Inflammation of the synovium
Fluid accumulation in the joint
Joint swelling and pain
If not diagnosed and treated vigorously–> destruction, deformity, disability

  • *Knee**
  • Synovitis causing Baker’s cyst (rupture-DVT)

Epidemiology:
Relatively common disorder
- Affects >1% of the population of North America
- Female:male 2.5:1
- Incidence increases with age (Peak: 4th-6th decade)

Associated with shortened life expectancy
Constitutional symptoms: LGF, wt loss, anorexia, fatigue
Extra-articular involvement:
- Lungs, skin, heart, hematologic, eye, vessels
- Rarely: CNS (rheumatoid nodules), kidneys
- Differentiate from drug side effect!

How well did you know this?
1
Not at all
2
3
4
5
Perfectly
16
Q

A 30 year old woman presents to her physician with worsening joint pain over the last 6 months, especially in her knees. She noticed some pain and swelling in her hands but thought it was related to her work in a dressmaker’s shop, but now her knees are hurting as well. Based on gross specimens below, what types of changes are occuring and what is her possible diagnosis?

A

Rheumatoid arthritis: Articular cartilage destruction from periphery of joint (central sparing

Below: Papillary synovial hyperplasia, subsynovial lymphoplasmacytic infiltration

  • *Pathophys**:
  • Overactive T cells–> attack synovial fluid space–> Synovial inflammation:
  • Fluid filled with neutrophils
  • Pannus infiltrated by T lymph, macrophages, fibroblasts, plasma cells, endothelial dendritic cells
  • -> chronic inflammation
  • –> Bone, cartilage, ligament, tendon damage
  • -> erosions, joint deformity

Treatment:
Goal is complete remission
1. Start with anti-inflammatory medications
- NSAIDs
- Corticosteroids
2. Add immunomodulators (DMARDs) early
3. If no significant improvement in first few months add “biologics”
- Antibodies against inflammatory cytokines

How well did you know this?
1
Not at all
2
3
4
5
Perfectly
17
Q

A 45 year old woman with a 20 year history of rheumatoid arthritis comes to the emergency room with the following necrosis in her fingertips. Additionally she has a marked footdrop. What is occuring and what is her treatment?

A

RA vasculitis: extremity and nerve involvement:
Overactive T-lymphocytes infiltrating and destroying capillary structures supplying fingertips, nerves
Vasculitis
- Arterioles, capillaries, venules
- Mostly leukocytoclastic vasculitis
- Skin ulcerations (necrotizing)

Nervous system:

  1. Myelopathy
  2. Mononeuritis multiplex:
    - disorder characterized by simultaneous or sequential damage to more than one nerve group
    - isolated damage to at least 2 separate nerve areas
    - involves destruction of the axon
    - interferes with nerve conduction at the location of the damage (see wrist, foot drop)

    - causes include a lack of oxygen caused by decreased blood flow or inflammation of blood vessels. No cause is identified for about one-third of cases

Treatment:
Goal is complete remission
1. Start with anti-inflammatory medications
- NSAIDs
- Corticosteroids
2. Add immunomodulators (DMARDs) early
3. If no significant improvement in first few months add “biologics”
- Antibodies against inflammatory cytokines

How well did you know this?
1
Not at all
2
3
4
5
Perfectly
18
Q

A 65 year old woman with a 40-year history of rheumatoid arthritis presents to her physician with the followin in her eye. What has occured? Additionally, she has been having increasing difficulty breathing. What might be causing her breathing problems?

A

Rheumatoid arthritis: extra-articular manifestations:

Front: Ocular
- Keratoconjunctivitis sicca (also seen in Sjogren’s)
- Episcleritis (more superficial than scleritis)
- Scleritis
- Scleromalacia perforans (rheumatoid nodule)
Below:
Pulmonary
- Pleural disease
- Interstitial lung disease
- Pulmonary nodules

How well did you know this?
1
Not at all
2
3
4
5
Perfectly
19
Q

Below is a histological specimen from the PIP joint of a 40 year old woman with a history of RA. What is present in her joint that is abnormal and how might it be contributing to her symptoms?

A

Rheumatoid Arthritis: Pannus:
Granulation tissue formation over joint space (arthritis) or over cornea, etc.
Pathophys:
- Synovial inflammation:
- Fluid filled with neutrophils
- Pannus infiltrated by T lymph, macrophages, fibroblasts, plasma cells, endothelial dendritic cells
- Leads to articular cartilage and juxta-articular bone loss

  • *Front**: Pannus overlying articular cartilage
  • *Back**: hyperplastic synovium of pannus
How well did you know this?
1
Not at all
2
3
4
5
Perfectly
20
Q

A 50 year old woman with a 30 year history of rheumatoid arthritis presents to her physician after noticing a nodule on her elbow that is giving her some pain. The nodule is biopsied and the following is found. What is it and why did it form?

A

RA nodule (extra-articular):
Subcutaneous nodules at areas of pressure (skin on elbows, legs)
- Moveable, firm, rubbery, occasionally tender

  • *Front**: rheumatoid nodule with central fibrinoid necrosis
  • *Below**: Palisaded macrophages surrounding rheumatoid nodule
How well did you know this?
1
Not at all
2
3
4
5
Perfectly
21
Q

The same 45 year-old caucasian female also complains that she has to drink water more frequently, not because she is thirsty, but because she is unable to swallow food or speak properly without doing so. Exam of her tongue revelas the following. What other oral manifestations would be seen in Sjogren’s?

A

Sjogren’s: oral involvement:
Dry mouth (xerostomia)–> decreased saliva
- Difficulty swalling food, inability to speak, change in taste, burning sensation, increased in dental caries
- Physical exam: dry, erythematous, sticky oral mucosa, dental caries, scanty/cloudy saliva, angular chelitis
- Parotid, major salivary gland enlargement in 60% patients (episodic or chronic, unilateral–> bilateral)

Complications:

  • accelerated caries, loss of dentition, poor fitting dentures,
  • oral candidiasis (yeast infection with Candida albicans),
  • sialolithiasis (salivary gland stones),
  • sialostenosis (salivary gland strictures),
  • bacterial sialadenitis (infection),
  • disturbed sleep (related to nocturnal fluid ingestion),
  • depression and weight loss (related to oral discomfort, dysphagia etc.).

Diagnostic questions:

  1. Symptoms of dry mouth for at least 3 months?
  2. Recurrent or persistently swollen salivary glands?
  3. Need for liquids to swallow dry foods?

Objective documentation of dry mouth

  • Oral examination: diminished salivary pool, loss of glistening of tongue, mucous membranes, erythema, fissures on tongue
  • Sialometry: Measure salivary flow using Lashley cup; circular cup connected to plastic tube, collect parotid saliva (placed over Stensen’s duct)- normal resting = 0.1 ml/min, after citric acid = 0.5-1.5 ml/min
  • Salivary scintigraphy: nuclear medicine test to measure isotope uptake into glands (saliva formation) plus stimulated salivary discharge
  • Sialography: inject contrast medium into salivary ducts followed by routine x-rays (MRI)

Confirmation of histologic involvement by salivary gland biopsy (below)

  • Labial minor salivary gland biopsy on inner aspect of lip
  • Findings: focal lymphocytic sialadenitis= accumulation of 50+ mononuclear cells around salivary gland acini, ducts
    1. Divide number of foci by glandular surface area= focus score
    2. Focus score > 1/4 mm3= Sjogren’s
How well did you know this?
1
Not at all
2
3
4
5
Perfectly
22
Q

A 45-year old caucasian female comes to her physician complaining of persistant dry eyes. On exam it is noted that her eyes appear red and there is destruction of the conjunctival epithelium. What other tests can be done to test for pathological dry eyes and what other symptoms would indicate Sjogren’s Syndrome?

A

Sjogren’s keratoconjunctivitis, dry eye:
Chronic inflammation of lacrimal glands:
- produce decrease in tear production
- Diminished tear production leads to the destruction of both corneal and conjunctival epithelium and a constellation of clinical findings termed keratoconjunctivitis sicca (KCS).

Physical signs:
- Dilation of the conjunctival vessels, pericorneal injection, (red eye)
- Irregularity of the corneal image and
lacrimal gland enlargement.
- The patient usually complains of a burning, sandy or scratchy sensation under the lids, itchiness, redness and photosensitivity.
- Failure to treat dry eyes may result in complications such as corneal ulcers or melting, corneal perforation, loss of vision, bacterial conjunctivitis or blepharitis (inflammation of the eyelids).

Diagnostic questions:

  1. Symptoms of dry eyes for at least 3 months?
  2. A foreign body sensation in the eyes?
  3. Use of artificial tears 3 or more times per day?

Objective documentation of dry eyes

  • Schirmer’s test (below): measures tear production on strip of paper in eye duct (normal > 10mm/5 min)
  • Rose Bengal/ fluorescien corneal staining: ocular surface irregularity or damage due to dryness
  • Tear break film test: tear film stability between last blink and appearance of dark, non-fluorescent areas in tear film (normal > 10 seconds)
  • *Diagnosis of Sjogren’s syndrome**:
    1. Dry mouth: documentation
    2. Dry eyes (document via above tests)
    3. Autoantibody presence:
  • Positive ANA, Rheumatoid Factor (> 1:160)
  • Positive anti-SSA (Ro) or anti-SSB (La)
    4. Salivary gland biopsy for histologic involvement
How well did you know this?
1
Not at all
2
3
4
5
Perfectly
23
Q

A 40 year old obese man comes to his physician because he developed a recent onset of excruciating pain in his left big toe (below). This has never happened to him before and there was no trauma to his foot. What might his diagnosis be and how would this be confirmed?

A

Acute gouty attack:
Acute attack
- Abrupt, monoarticular
- Involves MTP –> ankle, tarsal joint, knee, wrist, elbow
- Exquisite tenderness, swelling, warmth
- Fever, leukocytosis may occur
- Overlying skin desquamates
- Lasts 3-10 days w/o treatment

Precipitating event of acute attack:

  • Trauma, surgery, stress, infection, food (red meat, shellfish), alcohol (beer)
  • Diuretics, low dose aspirin
  • *Diagnosis of Gout: Urate crystals:**
  • Crystal in the aspirate:
  • Monosodium urate monohydrate= needle shaped
  • Differentiate from calcium pyrophosphate dihydrate (pseudo-gout)- NOT needle shaped
  • Crystal aligned with polarizer= yellow
  • Perpendicular to polarizer= blue
  • Negative birefringence (vs CPP= positive birefringence)
  • Crystal inside neutrophils= acute gout attack
How well did you know this?
1
Not at all
2
3
4
5
Perfectly
24
Q

A 60 year old man with a history of recurrent gout comes to his physician who notes the following on his ear. What is this a symptom of and how is the diagnosis confirmed?

A

Chronic Gout tophi: deposition of urate crystals (below)

Acute arthritis; inflammatory response to MSU (urate) crystals in joints

  • Generally starts in 1st MTP joint (big toe)
  • Peripheral joints, tendons, any tissue
  • Recurent
  • Chronic, deforming
  • Males/post-menopausal women
  • Associated with Lesch-Nyhan syndrome: mutations in the HPRT gene located on X chromosome–> HGPRT enzyme defect–> uric acid buildup

Pathophys:

  • Sodium urate triggers immediate immune response (potent trigger)
  • Leads to production of IL-1–> inflammatory cascade
  • Cells move to site of urate deposition
  • Cannot remove urate
  • Eventually immune response tapers

Clinical presentation of Chronic gout (tophaceous gout):
Result of:
- Years of persistent hyperuricemia
- Frequent attacks
- Inadequate therapy

Features of chronic gout:

  • Joint destruction
  • Tophi: subcutaneous tissue, tendons, IP-MCP joints, pinna of ear
  • Extra-articular: myocardium, pericardium, aortic valves, extra-dural spinal, kidneys
  • May mimic RA but less symmetrical
How well did you know this?
1
Not at all
2
3
4
5
Perfectly
25
Q

A 35 year woman comes to her physician after vacationing with her family because she developed a horrible rash on her face and neck. What is possible cause of the rash and how frequently is this the first manifestation of this disease?

A

SLE: cutaneous manifestations

  • *Front**: photosensitivity rash (both cutaneous and overall ill-feeling when exposed to sunlight)
  • *Below**: Malar rash: Fixed erythema, flat or raised, sparing the nasolabial folds

SLE: primary manifestations:

  1. Arthritis (55%)
  2. Skin involvement (25%)
  3. Nephritis (5%)
  4. Fever (5%)
  5. Other (15%)
How well did you know this?
1
Not at all
2
3
4
5
Perfectly
26
Q

What is the phenomenon noted below in a patient diagnosed with systemic lupus? What is the epidemiology of her disease?

A

SLE- Jaccoud’s arthropathy:

Early onset: Finger alignment appears abnormal but can be manipulated back into normal position
Later onset: permanent deformity

  • Incidence of 1.8 to 35 cases per 100,000/ year in general population
  • Prevalence fluctauates from 250000 to 1.5 million cases of lupus.
  • Women>Men – 9:1 ratio
  • African Americans>Hispanics>Whites
  • Usual onset between 15-45 years (60% cases)
How well did you know this?
1
Not at all
2
3
4
5
Perfectly
27
Q

Below is a patient being seen for Systemic lupus. What type of deformity is apparent and what caused these changes?

A

SLE: discoid rash: Raised patches, adherent keratotic scaling, follicular plugging; older lesions may cause scarring

Etiology is unknown
• SLE is an autoimmune disease
• Genetic factors
– HLA association (HLA DR2/HLA DR3)
• Environment factors
– Sunlight, drugs, chemicals, foods
– Bacterial or viral infections
– More associated with flares and exacerbations
• Hormonal factors
– Increased estrogen and prolactin levels
- Flares during pregnancy (vs RA)

Immune changes:
- Abnormal complement components (C1q, C4, etc )
- Fc Receptor abnormalities
–> delay in clearing dying (apoptotic) cells, exposing more autoantigens.

How well did you know this?
1
Not at all
2
3
4
5
Perfectly
28
Q

A 16 year old girl with SLE is brought to her physician because of a new onset of rash and papules in her mouth. Her mother is worried about the progression of her daughter’s condition and wants to know what other manifestations may occur. What might happen in a lifetime of SLE?

A

SLE: bullous lesions
Joints 90%:
- Arthritis and/or artharlgias are the most common presenting manifestation of SLE
- Most cases: Symmetrical.
- Unlike RA, is usually non-erosive ( at xrays) w/ subluxations, initially reversible.
- Involve periarticular surfaces.
- Increase frequency of tendon rupture.
- Fibromyalgia can overlap joint sx.

Skin
- Rashes 70%: butterfly rash, interarticular dermatitis, palpable purpura
- Discoid Lesions 30%: erythematous, hyperpigmented; flattened; scarred central areas with thin/fragile epidermis; follicular plugging
** Discoid lupus can become systemic lupus
- Alopecia: diffuse 40%

Pleuropericardium 60%

  • Serositis: pleuritic pain more frequent than effusions
  • Fluid exudative with normal glucose

Pulmonary manifestations:

  • Pneumonitis
  • Pulmonary hemorrhage (small capillary leak)
  • PAH
  • Shrinking lung syndrome (diaphragm, respiratory muscles)

Cardiac manifestations:

  • Most common cause of mortality in SLE is CAD
  • Endocarditis in 15-60% (asymptomatic)= Libman-Sacks endocarditis
  • Myocarditis in 9%: mild, seen as diastolic dysfunction
  • Clots in placental tissue–> miscarriages

Kidney 50%
Raynaud’s 20%
Mucous Membranes 15%
CNS (Seizures/Psychosis) 15%

How well did you know this?
1
Not at all
2
3
4
5
Perfectly
29
Q

A 50 year old woman has been suffering from SLE for 20 years and comes into her physician with increasing problems breathing. An MRI reveals the following. What is occuring with her lungs?

A

SLE: pulmonary manifestations
“Shrinking lung syndrome”
Pulmonary manifestations of SLE:
- Pneumonitis
- Pulmonary hemorrhage (small capillary leak)
- PAH
- Shrinking lung syndrome (diaphragm, respiratory muscles)

How well did you know this?
1
Not at all
2
3
4
5
Perfectly
30
Q

A 45 year old man with a 20-year diagnosis of SLE comes to his physician for a physical. His urine sample reveals elevated protein levels. Based on the histology below, what is his diagnosis and what types of changes are seen in kidneys in patients suffering from lupus?

A

SLE: lupus nephritis
Class I: Normal glomeruli
a) Nil by all techniques
b) Normal by light but deposits on EM or IF

Class II: Mesangial glomerulonephritis: mesangial cells affected

Class III: Focal glomerulonephritis: endothelial cells affected

Class IV: Diffuse glomerulonephritis: endothelial cells affected

Class V: Diffuse membranous glomerulonephritis: epithelial cells affected

Class VI: Advanced sclerosing glomerulonephritis

Lupus nephritis:
Anti-DNA Antibody deposition, complement activation will produce inflammation, occlusion and disruption of glomeruli that translate in:
- Protein urinary excretion (Nephrotic Sd)
- Hematuria (Nephritic Sd)
- Increase on Creatinine levels
- Renal Failure

How well did you know this?
1
Not at all
2
3
4
5
Perfectly
31
Q

A 65 year old woman with a history of SLE expired from a Myocardial ischemic attack. On autopsy the following is found in her coronary valves. What has occured and did this cause her heart attack?

A

SLE: Libman Sack’s Endocarditis:
Cardiac manifestations:
- Most common cause of mortality in SLE is CAD
- Endocarditis in 15-60% (asymptomatic)= Libman-Sacks endocarditis
- Myocarditis in 9%: mild, seen as diastolic dysfunction
- Clots in placental tissue–> miscarriages

How well did you know this?
1
Not at all
2
3
4
5
Perfectly
32
Q

A 55 year old man with a 20 year history of smoking 2 packs a day is diagnosed with small cell lung carcinoma. 3 weeks after his diagnosis he develops a rash on his face. His oncologist believes it is related to his treatment but the patient also complains of exercise intolderance and muscle pain. A muscle biopsy reveals the following. What is his diagnosis and what was the pathogenesis of this disease?

A

Dermatomyositis
Afflicts children and adults
- Characteristic rash affecting the upper eyelids, face, and trunk
- Increased association with an underlying malignancy (usually carcinoma) in males
- Polymyositis and inclusion body myositis have a much reduced risk of underlying malignancy compared to dermatomyositis in adults.

Pathogenesis:

  • Immune complex formation with IgG, IgM and complement including C5-9 membrane attack complexes in the walls of blood vessels
  • NOT direct attack on muscle; also affects dermis as vascular attack occurs here too

Histology:

  • *- Microangiopathy with loss of capillaries,
  • **perifascicular atrophy****
  • Perivascular infiltrates of B and T cells, predominance of CD4+ T cells
  • Muscle infarcts
  • Skin rash also related to the microangiopathy
  • All fibers at edge of fascicle shrivel up
  • Look for complement deposition in capillaries
How well did you know this?
1
Not at all
2
3
4
5
Perfectly
33
Q

A 40 year old woman with Raynaud’s develops pain and weakness in her legs which is worse with exercise. A muscle biopsy below reveals lymphocytes within her muscle fibers causing damage. What is her diagnosis?

A

Polymyositis:
Inflammation focused on the muscle fibers themselves:
- No microangiopathy as in dermatomyositis
- Perifascicular atrophy absent
- CD8+ T cell mediated cell injury
- Frequent association between anti-Jo-1 (recognizes a histidyl-tRNA synthetase) as well as interstitial lung disease, Raynaud phenomenon, and nonerosive arthritis

Below: necrosis and regeneration of muscle tissue

How well did you know this?
1
Not at all
2
3
4
5
Perfectly
34
Q

A 45 year old man presents to your office due to a new onset of pain and cracking at the edge of his lips that he has tried treating at home with neosporin but won’t go away. Additionally he has a 5-year history of HIV infection. Based on his clinical presentation (below), what is his diagnosis and what would be the histological presentation?

A

Oral candida infection
The most common fungal infection in the oral cavity and oropharynx
- In many cases other factors play a role: antibiotic therapy, immunosuppression, xerostomia, anemia

Clinical presentation: varies : asymptomatic, pseudomembranous (thrush), erythematous, angular cheilitis (red, scaling, fissuring area at the corners of the mouth)

Histopath:
Fungal hyphae or pseudophypahe and ovoid spores (PAS positive) associated with neutrophilic microabscess
On exfoliative cytology or tissue sections

How well did you know this?
1
Not at all
2
3
4
5
Perfectly
35
Q

A 36 year old man presents to his physician after experiencing fever and general malaise over the past 6 days. On physical examination, a blister is noted on his lip that he states appeared around the time of his feeling ill. What does he likely have and what would be visible on histology?

A
  • *Herpes simplex virus 1 infection**:
  • First exposure to HSV1: 90% of patients have subclinical symptoms, 10% of patients develop primary herpetic gingivostomatitis

Herpetic gingivostomatitis: fever, malaise, lymphadenopathy, mucosal erythema, vesicles and ulcers that resolve in 7 to 14 days.

  • *Latent virus** within trigeminal ganglion reactivates and results in infection of the vermilion border of the lip (herpes labialis, “fever blister”, “cold sore”)
  • Cluster of fluid-filled vesicles which rupture, crust and heal within 7-10 days.

Histopath:
Nuclear enlargement of infected cells
Condensation of the chromatin at the periphery of the nucleus
Multinucleated cells formed by fusion of infected cells
Inflammation and edema of adjacent mucosa

How well did you know this?
1
Not at all
2
3
4
5
Perfectly
36
Q

A 33 year old woman goes to her physician for a sore throat. On exam, there are abscesses noted in her tonsillar crypts. Histologic exam of a specimen reveals the following. What is her diagnosis?

A

Actinomyces infection:
Saprophitic, Gram positive anaerobic bacteria part of the normal flora
- Colonize tonsillar crypts, dental plaque, gingival sulci

Acute form: painful abscess

Chronic form: extensive fibrosis (hard, wooden induration), fistula formation, periostitis, osteomyelitis

Histo: Colonies of club-shaped filaments arranged in a rosette pattern surrounded by neutrophils
Granulation tissue

Can occur in patients after radiation therapy–> chronic osteomyelitis due to actinomyces infection

How well did you know this?
1
Not at all
2
3
4
5
Perfectly
37
Q

A 50 year old man is referred to an oral maxillofacial surgeon after finding the following abnormality in his mandible on X-ray at the dentist. The abnormal tissue is ressected and shows enamelly-derived tissue and odontogenic epithelium. What type of tumor is it and what is the common treatment?

A

Ameloblastoma:
Tumor arises from enamel organ or progenitor cell lines
- Develop soft tissue components of odontogenic epithelial
- Benign, slow-growing, locally inasive
- Arise in mandibular ramus, maxilla, floor of nasal cavity

Histo:

  • Islands of proliferating odontogenic epithelium reminiscent of enamel organ
  • Basophilic columnar cells at periphery with reverse polarization (nuclei away from basement tissue)
  • Edematomous cells in nests or cords

Treatment:

  • Wide surgical ressection
  • Long-term follow up due to 25-55% recurrence rate
How well did you know this?
1
Not at all
2
3
4
5
Perfectly
38
Q

A 35 year old woman presents to her physician after a dentist appointment where the dentist discovered a mobile mass in her left parotid gland. The mass was excised and the following was discovered. What is the most likely diagnosis and treatment? What is the prognosis?

A
  • *Pleomorphic adenoma of salivary gland**
  • MOST COMMON salivary glands neoplasm
  • Most commonly in the PAROTID followed by minor salivary glands
  • Wide age at presentation
  • PAINLESS, slowly growing mass, mobile
  • Paresthesia due to nerve compression - rare

Histo:

  • Well-circumscribed, variably encapsulated mass
  • Multinodular (esp. in recurrent disease)
  • Composed of three cell types:
    1. Epithelial glandular ductal structures
    2. Myoepithelial cells
    3. Mesenchymal stroma: myxoid, chondroid, hyalinized, osseous
  • *Treatment:**
  • Surgical excision
  • *Prognosis**:
  • Recurrence rate: 45% after simple enucleation (due to lack of encapsulation or incomplete removal)
  • Recurrence rate: 2.5 % with superficial or total parotidectomy
  • Malignant transformation – in 2-7%
How well did you know this?
1
Not at all
2
3
4
5
Perfectly
39
Q

The following histologic specimen was obtained from a parotid gland mass. There was a similar mass on both parotid glands (bilateral). Based on the appearance, what is the diagnosis, treament and prognosis?

A
  • *Warthin’s tumor**:
  • Second most common benign salivary gland tumor
  • Characteristically presents in the parotid
  • Bilaterality or multifocality occurs more frequently than any other tumor, and it is synchronously identified with other tumors more than any other tumor type
  • *Histo:**
  • Circumscribed, solid/cystic. Cysts may contain yellow-brown fluid.
  • Papillary fronds and cystic spaces lined by double layered oncocytic epithelium
  • Dense lymphoid node-like stroma intimately associated with the epithelial component
  • *Treatment:**
  • Surgical excision
  • *Prognosis:**
  • 4-25% recurrence due to multifocality or incomplete excision
How well did you know this?
1
Not at all
2
3
4
5
Perfectly
40
Q

A 46 year old man presents to his physician for a physical. He notes a recent development of a painless blister on the inside of his lip. He thinks it may be related to having accidentally cut his lip while eating nuts a few weeks ago. What is his diagnosis and treatment?

A

Salivary Mucocele
The most common non-neoplastic lesion of salivary glands
- “Pooling of mucin in a cystic cavity”

Types:

  1. Retention type – mucin pooling confined within a dilated excretory duct
  2. Extravasation type – escape of mucin from the duct system into connective tissue – most common type

Seen in: lower lip, tongue, floor of mouth, palate

Presentation: soft, fluctuant semitranslucent painless swelling that may occur after a traumatic event

Treatment - complete excision, including minor salivary gland

How well did you know this?
1
Not at all
2
3
4
5
Perfectly
41
Q

A 47 year old woman presents to her doctor because of increasing issues with her mouth and eyes. She says she feels like her eyes are dry all the time, even when she uses eye drops, and she has to drink lots of water to be able to swallow any food because she doesn’t make enough saliva. Based on these two symptoms and the biopsy of one of her salivary glands (below), what is her diagnosis?

A

Sjogren’s salivary gland:
Describes a characteristic lymphocytic infiltrate in salivary glands; doesn’t commit to an etiologic cause but describes histologic findings

Most cases – women in the 4th and 5th decades
- Associated with or is a precursor of Sjogren’s syndrome

  • *Two types:
    1. Systemic: collagen vascular disease
    2. Localized to salivary and lacrimal glands**

Pathology:
- Uni or bilateral enlargment of parotid glands with preserved lobulation

Histo:

  • Periductal chronic inflammation extending into acini
  • Glands replaced by polyclonal lymphocytes, germinal centers, plasma cells
  • Lesion= lymphoepithelial sialadenitis (epimyoepithelial islands)
  • Similar changes present in minor salivary glands in Sjogren syndrome (biopsy smaller glands first)- need clinical diagnosis and serologic specimen to confirm dx
How well did you know this?
1
Not at all
2
3
4
5
Perfectly
42
Q

A 56 year old woman complains of a recent onset of pain in her cheek. The pain is exacerbated by eating or smelling food. An MRI reveals a calcification in her salivary gland and the following is removed. What is in her salivary gland?

A

Salivary gland sialolithiasis:
Collection of concretions which form a stone within the salivary gland excretory duct system (stone in duct)
- Most common involves the submandibular gland (Wharton’s duct) and may cause a chronic sialadenitis distal to the stone.
- Always visible on radiographic examination
- Nidus of cellular debris in the center of concentric laminated calcium deposits

How well did you know this?
1
Not at all
2
3
4
5
Perfectly
43
Q

Below is a specimen from the nasal sinuses of an otherwise healthy 23 year old man. He complains of recent worsening of his seasonal allergies after avoiding going outside in the spring. Based on the histology, What is his diagnosis?

A
  • Hypersensitivity reaction to fungal antigens (atopic patients), aspergillus
How well did you know this?
1
Not at all
2
3
4
5
Perfectly
44
Q

A 54 year old man with a history of chronic recurrent sinusitis presents to an ENT to have his sinuses examined. A ball of material is removed from his sinuses and examined under the microscope. What is his diagnosis?

A

Mycetoma
Fungus ball (mycetoma, aspergilloma)
- Sinuses with poor drainage; fungi proliferate and form dense mass of hyphae
- DO NOT invade mucosa/vessels

How well did you know this?
1
Not at all
2
3
4
5
Perfectly
45
Q

A 65 year old man is admitted to the ICU after presenting to the ER with an infection on his face that had worsened over the past day. He has a history of poorly-controlled diabetes. A sample from the infected site was obtained and revealed the following. What is his diagnosis?

A
  • *Mucor**
  • Seen in immunosuppressed patients
  • Fungi invade sinus mucosa, vessels
  • Can spread to venous sinuses, meninges, brain
  • High mortality
  • Mucormycosis: common in diabetic patients; can involve skin, bone, orbit, brain
46
Q

A 46 year old woman presents to her physician with increasing sinus pressure related headaches. Recently her runny noses started to show some blood. She’s worried she may have an infection or tumor. The following gross specimen is removed from her sinuses. What is her diagnosis and prognosis?

A

Inverted papilloma:
Location: lateral nasal wall, middle meatus, paranasal sinuses, rarely in the nasopharynx or middle ear

Clinical presentation: nasal obstruction, epistaxis, rhinorrhea, facial pressure

CT, MRI – unilateral polypoid mass filling the nasal cavity, +/- displacement of nasal septum and opacification of sinuses

Histo:

  • Markedly thick , inverted growth pattern of neoplastic proliferation replacing mucoserous glands and ducts
  • Transitional/squamoid epithelium with numerous intraepithelial microcysts containing cellular debris
  • Stroma: edematous, myxomatous, fibrous. No seromucinous glands in the stroma
  • *Prognosis:**
  • Recurrence: up to 60% of cases (depending on type of surgery)
  • Malignant transformation (squamous cell carcinoma): in 10% of cases
47
Q

A 16 year old boy is brought to the doctor due to difficulty breathing through his nose. On endoscopic exam a mass is found in the posterior nasal wall. His physician is concerned and wants to have it removed as soon as possible. Why is he so concerned- what is his diagnosis and what are the risks of allowing the mass to remain in place?

A

Nasopharyngeal angiofibroma

  • Benign, highly cellular and richly vascularized mesenchymal neoplasms that arise in the roof of the nasopharynx
  • Exclusively in **YOUNG MALES (mean age 15yo)
  • There is an association with FAP**

Presentation:

  • Nasal obstruction, spontaneous epistaxis, nasal drainage
  • Endoscopy – mass involving the posterior nasal wall

Treatment:
- Preop angiography – identifies the feeding vessel and allows for presurgical embolization (reduces risk of surgical hemorrhage

Histo:

  • Round, nodular nonencapsulated masses with a sessile or pedunculated base
  • Tumor surface – covered by intact mucosa (+/- focal ulceration)
  • Microscopic: 3 elements:
    1. **abnormal vascular network (lack muscular wall- can’t constrict, can cause hemorrhage)**,
    2. connective tissue stroma (fine and coarse collagen)
    3. stromal cells (cytologically bland spindle cells)
  • *Treatment:**
  • Selective angiographic embolization, surgical excision
  • **Local aggressive growth, recurrences in 20% of patients, usually within the first 2 years after diagnosis
  • Mortality – up to 9%, due to hemorrhage**
48
Q

A 29 year old woman with a life-long history of otitis media is now suffering from hearing loss. A biopsy is taken from her superior posterior middle ear that reveals the following. What is her diagnosis?

A

Cholesteatoma:
Destructive squamous epithelial cyst of middle ear or mastoid region
- usually secondary to chronic otitis media, but occasionally congenital
- Origin in superior posterior middle ear, may demonstrate locally aggressive growth into adjacent structures

Presentation:
- May lead to progressive conductive hearing loss

49
Q

A 72 year old man with a 20-year history of poorly controlled diabetes presents to the Emergency room due to a worsening infection in his ear (below). He says there was no direct trauma, though he reports using a q-tip to clean his ears on a weekly basis. Based on the appearance, what is the likely diagnosis, treatment, and danger of not treating?

A

Malignant external “otitis”:
Infection of the external auditory canal which progresses to a highly aggressive, invasive osteomyelitis of the temporal bone and skull base, most often in immunocompromissed patients

> 90% of cases: in **DIABETICS, most over 60 yo

  • Initiated by minor trauma** (cleaning ear) to the external auditory canal mucosa that allows entry of an opportunistic agent (Pseudomonas aeruginosa)
  • Can lead to invasive osteomyelitis of skull base

Clinical presentation:

  • Erythema of the auricle and ear canal, otalgia
  • Severe headache, purulent otorrhea
  • Clinical exam: granulation tissue on the floor of the external auditory canal

Histo:

  • necrotic material and granulation tissue
  • Culture of debridement specimen is crucial in the management of MEO

Treatment:
strict diabetes control, careful debridement and IV multidrug antibiotic treatment

50
Q

Specimen from middle ear:

A
  • *Jugulotympanic ganglioma**:
  • The most common benign tumor of the middle ear
  • Slow growth (years)
  • Cells with lobular arrangement; rich, vascular stroma
51
Q

A 25 year old woman presents to her physican for a consult on having a mass removed from her neck. It is painless and has been there for several months. The physician performs a biopsy that reveals the following. What is her diagnosis and prognosis?

A

Branchial cleft anomaly:
Lateral cervical cyst that results from congenital/developmental defects arising from the primitive 2nd branchial apparatus
- 80-90% of branchial anomalies are associated with the second apparatus
- Lateral neck, near the mandibular angle
- Equal gender distribution, 75% of patients between 20-40 yo

Clinical presentation

  • Mass along the anterior border of the sternocleidomastoid muscle
  • Painless, long duration
  • May become secondarily infected

Prognosis, treatment:

  • Complete excision
  • Recurrence rate 2.7%

Histo:

  • Unilocular cysts, 2-6 cm
  • Lining: stratified squamous epithelium (90%), respiratory epithelium, or both
  • Keratinaceous debris inside the cyst
  • Lymphoid aggregates with or without germinal centers beneath the epithelial lining
  • Acute and chronic inflammation, foreign body giant cell reaction and fibrosis
52
Q

A 68 year old woman presents to her physician with knee pain. The pain has flared a few times over the past several years, but this this flare is more severe. An x-ray is taken that reveals calcifcation of the menisci of her knee. What is her diagnosis based on the x-ray and how would it be confirmed?

A
  • *Chondrocalcinosis**: meniscal calcifcation
  • Accumulation of CPPD in synovial membranes (pseudogout), joint cartilage (chondrocalcinosis), ligaments and tendons
  • It can be idiopathic, a/w trauma, linked to a number of metabolic disorders, or, rarely, hereditary

Pseudo-Gout= Degenerative Joint Disease (DJD)

  • calcium pyrophosphate crystals in articular, peri-articular tissue
  • Idiopathic or associated with certain metabolic diseases
  • *- Acute inflammatory process
  • 1 or more joints
  • Lasts for several days**
  • Similar to gout, not as severe
  • Smaller, less painful attacks between flares
  • Any joints, but knee most often
  • Crystal deposition in tendon, ligaments, synovium, cartilage
  • *- More common in elderly, women**
  • *Laboratory Findings
  • rod shaped, often intracellular crystals**
  • *- positively birefringent (blue, when parallel to the axis of a polarizing microscope compensator)**

Evaluation:
serum Ca, Mg, phosphorus, PTH, alkaline phosphatase, ferritin, iron, iron binding capacity, glucose, (TSH), uric acid

Radiologic findings:

  • punctate and linear densities in articular cartilage
  • Site: knee, acetabular labrum, symphysis pubis, articular disk of the wrist, and annulus fibrosus of the intervertebral discs

X-ray screening:
- AP views of both knees, pelvis, hips, symphysis pubis,
PA view of both hands.

53
Q

A 40 year-old woman presents to the physician with a new onset of pain and tingling in her fingers. She states it is often preceded by cold temperatures and it feels like the blood drains from her hands (below). A blood test reveals anti-topoisomerase 1 (Anti-Scl-70) antibodies. What is her possible diagnosis and how could this disease progress?

A
  • *Systemic sclerosis: Raynaud’s phenomena**
  • Anti-Scl-70 antibodies= seen in 30-40% Systemic sclerosis, diffuse type
  • Anti-centromere antibodies= 80% specific for Systemic sclerosis, more limited disease type

Raynaud’s phenomenon is the initial symptom of systemic sclerosis in a majority of patients.

  • Often precedes the onset of other SSc features.
  • Ultimately develops in virtually all SSc patients.

Is common among premenopausal women, only a minority of whom develop a connective tissue disease, most commonly SSc.

Characterized by abrupt, temporary occlusion of peripheral arteries.

  • Triggered by cold exposure, tobacco smoke or emotional stress.
  • Typical findings are the waxy blanching of fingers followed by numbness, tingling and paresthesias.
  • Restoration of blood flow causes cyanotic discoloration followed by erythema of affected regions.

Irregular capillary beds in Systemic sclerosis exacerbate phenomenon

  • *SSc-specific antibodies:**
    1. Anti-centromere antibodies > 80%
  1. Anti-Scl-70 antibody (anti-topoisomerase 1)= specific for SSc, present in 30-40% of patients
    - Seen more commonly in severe interstitial lung disease

** Both anti-centromere and anti-Scl-70 antibodies helpful in classification of SSc subset (anti-centromere= limited disease, Anti-Scl-70= diffuse)

54
Q

A woman comes to the physician for treatment for her scleroderma. What types of changes have occured in her hands? What other changes might be visible on imaging studies?

A

Systemic Sclerosis: Skeletal involvement
X-rays show resorption and often complete dissolution of the distal end of the phalanx (acro-osteolysis).

Indolent myopathy with minimal elevation in serum muscle enzymes (focal muscle fibrosis and fiber atrophy).

Some patients develop typical inflammatory myopathy (polymyositis) with elevation in muscle enzymes and EMG changes.

55
Q

A 45 year old patient with a history of Reynaud’s comes to her physician complaining of pain right below her sternum and a new case of hoarseness. On endoscopy, the following is found. What might her diagnosis be, and what other manifestations could be present?

A
  • *Scleroderma: GI involvement**
    1. Esophageal abnormalities in up to 90% of patients:
  • Incompetence of the lower esophageal sphincter and decreased or absent peristalsis in the lower two-thirds (smooth muscle involvement- NOT striated) cause dysphagia and odynophagia.
  • Chronic reflux and esophageal stasis cause esophagitis, Barrett`s esophagus and lower esophageal stricture.
  • Increased frequency of esophageal cancer in SSc.
  • Reflux of gastric contents causes hoarseness, cough and aspiration pneumonitis.
  1. Gastric/esophageal dilation
  2. Watermelon stomach= Gastric Antral Vascular Ectasia (GAVE) with painless severe GI bleeding
  3. Disordered motility:
    - Gastric distension
    - Bacterial overgrowth, diarrhea, malabsorption, pseudo-obstruction
    - Colonic disease with wide-mouth sacculation (pseudodiverticula)
56
Q

What types of lung changes occur in systemic sclerosis?

A

Scleroderma: pulmonary involvement:
Lung involvement is the leading cause of morbidity and mortality in SSc.

Lung involvement results in pulmonary fibrosis (ILD) or pulmonary hypertension.

Pulmonary fibrosis in SSc is insidious at onset and progressive.

Physical exam discloses fine inspiratory crackles at the lung bases.

Pulmonary hypertension and cor pulmonale may develop in patients with limited SSc, even in the absence of pulmonary parenchymal involvement.

Diagnosis:

  • Chest roentgenograms are insensitive for detection of early lesions.
  • Pulmonary function studies are more sensitive (decreased DLCO, decreased TLC).
  • Isolated reduction of DLCO (TLC/DLCO ratio > 1.5) suggests pulmonary hypertension.
  • High resolution CAT scan is highly sensitivity and allows the detection of alveolitis (ground glass appearance) in early stages.
  • Late CAT scan changes include severe fibrosis and traction bronchiectasis.

Histo: Loss of alveolar sacs due to fibroblast, inflammatory cell infiltration

57
Q

Below is the kidney recovered from a woman with systemic sclerosis. What types have changes have occurred grossly and microscopically? What is the greatest risk of these changes and in what types of scleroderma patients is it seen most frequently?

A

Systemic sclerosis- renal changes

Gross appearance: edema at surface, flea-bitten appearance due to glomerular hemorrhage, necrosis

Renal involvement is considered to be the most deadly complication of SSc.

Renal crisis= develops in 15-20 percent of patients and usually occurs with an abrupt onset (within hours)

Malignant hypertension, acute cardiac failure, myocardial infarction, stroke, and rapidly progressive renal insufficiency characterize scleroderma renal crisis.

Progression to renal failure and death occurs rapidly unless the diagnosis of renal crisis is established and aggressive treatment instituted promptly.

Patients with diffuse and rapidly progressive skin involvement are at the highest risk.

Histo: Renal arterioles with “onion skin” proliferation–> prevents diffusion across walls of vessels

58
Q

A 60 year old patient expired after a diagnosis of systemic sclerosis. Below is her heart at autopsy. What types have physical changes have occured, what types of pathological changes would this cause to cardiac function, and what other organ systems may have been affected?

A

Scleroderma: cardiomyopathy

Heart involvement: 80% of patients

  • clinically significant primary cardiac dysfunction is less common.
  • Tachyarrythmias and cardiac conduction disturbances are frequent.
  • Pericardial effusion is found by echocardiography in many patients, but symptomatic pericarditis is rare
  • Myocardium: replaced by fibers
  • Right heart strain in cases of pulmonary hypertension.

Histo: Pulmonary arteries: All organs affected by fibroproliferative tissue: lumen replaced by tissue–> dysfunctional vessels
respiratory failure, pulmonary HTN

59
Q

Below is a skin biopsy of a patient who recently developed Raynaud’s phenomena. Her physician detected anti-centromere antibodies in her serum. What types of histological changes have occured and what is her likely diagnosis based on the sample below?

A

Systemic sclerosis: cutaneous changes

Pressure atrophy due to thickening of skin–> loss of sweat glands, hair follicles

Skin sclerosis in 95% of patients. In 5% there is typical visceral involvement in the absence of skin sclerosis (Scleroderma sine Scleroderma).

Thickened and hidebound skin first appears distally (Sclerodactyly) and progresses proximally.

  • Flexion contractures at joints
  • Shiny, taut skin
    1. Limited cutaneous involvement:
  • hands, feet, face, forearms and legs.
    2. Diffuse cutaneous involvement:
  • upper arms, thighs, chest, abdomen and back in addition to above.

The face is expressionless with loss of wrinkles, beaked nose, thin lips with radial furrowing, and microstomia.

Diffuse hyperpigmentation or hyperpigmented hair follicles surrounded by depigmented areas (“salt-and-pepper” pattern).

Telangiectasia on the hands, face, lips and tongue.
- Mucosal telangiectasia (gastric, small intestinal or colonic, implicated as a source of gastrointestinal bleeding).

Painful ulcerations at the fingertips and other areas where the skin is tightly stretched.

Cutaneous calcifications at the fingertips, along the extensor surface of the forearms, and periarticular areas.

60
Q

Below is a thyroid biopsy from a patient diagnosed with systemic sclerosis. She has recently developed an increased intolerance for cold and has gained 15 pounds with no real changes in eating habits. Based on the biopsy sample, what changes have occured? What other types of visceral changes occur in systemic sclerosis?

A

Systemic sclerosis: Thyroid changes
Thyroid: dysfunction due to infiltration of colloid follicles–> hypothyroidism (detected in 1/3 of SSc patients)

  • *Peripheral neuropathy** (trigeminal neuralgia and carpal tunnel syndrome) can occur but the CNS is spared.
  • Can cause some pain

Erectile dysfunction develops in many male patients, and can precede other SSc manifestations.

The sicca syndrome is frequently found (Secondary Sjögren’s Syndrome).
- Replacement of exocrine glands by fibrotic tissue

Primary biliary cirrhosis has been documented in a number of cases.

61
Q

A 22 year old woman comes to the emergency room with a fracture of her ulna. An x-ray reveals the following. What is her diagnosis and treatment?

A

Fibrous dysplasia:
Developmental abnormality
Path: activating mutation in alpha subunit of guanine nucleotide binding protein–> increased cAMP

X-ray: lucent ground glass quality, well-marginated borders

Histo: disorganized fibrous and osseous elements
- Irregular trabecular bone within fibrous stroma, no osteoblastic component

  1. Monostotic:
    - Most common (75-80%)
    - 2nd to 3rd decade, M=F
    - Sites: proximal femur, tibia, ribs, facial bones
    - Asymptomatic or pathologic fractures
  2. Polyostotic:
    - 25%, 1/2 skeleton
    - Childhood, F > M
    - Pathologic fractures
    - Pregnancy may stimulate growth
  3. McCune Albright:
    - FD + endocrine, skin, sarcoma (< 1%)

Treatment: curettage, fracture repair

62
Q

A 6-year old boy was brought to the emergency room by his mother after he fell and hurt his leg. An X-ray reveals the following. A bone sample reveals spindle cells with multinucleated giant cells and macrophages. What is his diagnosis?

A

Non-ossifying fibroma:
AKA: Fibrous cortical defect
- Common (25-40%, 4-10 years old)
- Sites: tibia, femur
- Neoplastic vs developmental
- May see fratures

X-ray: cortical, scalloped, eccentric

Histo: spindle cells with multinucleated giant cells, macrophages

63
Q

A 16 year-old boy who broke his arm two years ago presents with another fracture in the same site on his arm. An x-ray reveals a demarcated area within his bone that was removed (below). What occured and what is his diagnosis?

A

Solitary bone cyst:
Unilocular, fluid-filled, non neoplastic cyst
- Disturbance of bone growth with trauma
- Kids/adolescents, M > F (3:1)
- 2/3 upper humerus, femur
- Metaphyseal, adjacent to growth plate

X-ray: demacated

Histo: thin, fibrous cyst wall

Clinical: fracture, usually asymptomatic

64
Q

A 17 year old girl had an x-ray performed on her arm after spraining her wrist. The following lesion was found. On surgical removal, large amounts of blood came out of the lesion. What is the lesion and why was it removed?

A

Aneurysmal bone cyst:
Expansive, hyperemic (blood filled sponge)
- Kids, adolescents
- Periosteum around lesion thin but intact

Histo: granulation tissue, mononucleated giant cells and osteoid trabeculae

May represent cystic degeneration of underlying lesion

Treatment: curettage (remove entire segment due to risk of osteosarcoma underlying hemorrhage

65
Q

A 20 year old man went to his family physician for treatment of swelling in his hand that worsened at night. Below is an image of his hand after treatment with injected steroids which failed to help his pain. He states the only thing that helps his pain is taking aspirin, but what might his diagnosis be and how should he be treated?

A

Osteoid osteoma:
Small, painful benign lesion
- M > F (3:1), 5-25 years
- Cortical, usually metaphysis; diaphysis of lower extremity long bones

Clinical: pain, worse at night, relieved by ASA

X-ray: sclerotic cortical lesion

Histo: nidus composed of thin trabecular bone, granulation tissue background
- Numerous osteoclasts, osteoblasts

Treatment: curettage, radiofrequency, electrocautery, mini bloc excision

66
Q

A 22 year-old man breaks his finger and has an abnormal x-ray which leads to a biopsy of the lesion. What changes have occured here and what is his diagnosis?

A

Osteoblastoma:

Histo: similar to osteoid osteoma but larger

  • M=F, 10-25 years
  • NOT accompanied by sharp pain

Sites: mainly spine, long bones

Imaging shows benign process: encapsulated, no inflammatory reaction

67
Q

A 45 year old woman breaks her arm and has a concerning x-ray that prompts the surgeon to remove a section of the bone in her arm. What is her diagnosis, concerns for future developments, and treatment?

A

Enchondroma:
Intraosseous tumor
- Well-differentiated cartilage
- Any age, asymptomatic
- Removal only due to concern of well-differentiated osteosarcoma

Site: metacarpals, phalanges
- Hands/ feet

X-ray: well delineated, lytic

Histo: increased cellularity, atypia

Treatment: curettage if painful or causing fractures

68
Q
A

Enchondroma

Intraosseous tumor

  • Well-differentiated cartilage
  • Any age, asymptomatic
  • Removal only due to concern of well-differentiated osteosarcoma

Site: metacarpals, phalanges
- Hands/ feet

X-ray: well delineated, lytic

Histo: increased cellularity, atypia

Treatment: curettage if painful or causing fractures

69
Q

A 15 year old girl fractures her humerus after falling down in a soccer game. The x-ray reveals a lytic lesion at the epiphyseal plate. A bone biopsy is performed that reveals the following (below). What is her diagnosis?

A

Chondroblastoma
Uncommon epiphyseal tumor
- 2nd-3rd decade, M > F

Sites: femur, tibia, humerus

X-ray: lytic, eccentric

Histo: chondroblasts, mineralized matrix, multinucleated giant cells; “Chicken wire” matrix

70
Q

A 66 year old man presents with bone pain in his shoulder. He has also had a 20 pound weight loss over the past 6 weeks. Blood testing reveals elevated plasma cells and an x-ray shows a lytic lesion in his humerus. What is his likely diagnosis, treatment, and prognosis?

A

Multiple Myeloma:
Malignant plasma cells
- M > F, 65 years
- Sites: skull, spine, ribs, pelvis, femur

X-ray: lytic

Plasmacytoma=
Multiple myeloma=

Histo: sheets of plasma cells

Diagnosis:
- Protein electrophoresis

Treatment: radiation, chemo

  • MM: Poor prognosis (32 months)
  • Plasmacytoma: 65% 5-year survival
71
Q

A woman that had previously fracture her ulna at the olecranon process developed a fast-growing tumor. Based on her history and the histology of the tumor, what is her diagnosis and treatment?

A

Nodular fasciitis:
Benign, fast-growing tumor
- adults
- Site: extensor surfaces

Etiology: trauma-induced reactive proliferation

Histo: active fibroblastic lesion

Treatment: excision

72
Q

A 45 year old diabetic woman comes to her physician with a bump on the palmar surface of her hand. An x-ray reveals a bony lesion. The lesion is removed and the following is revealed. What is her diagnosis and what put her at risk for developing this?

A

Fibromatosis:
Slow-growing, locally aggressive
- Increased incidence in diabetics, alcoholics, epileptics
- May recur after excision
- Palmar (1-2%), plantar

Histo: bland spindle cells

73
Q

A 55 year old man presents with a firm mass in his distal femur above his patella. A bone biopsy revelas the following “herringbone” patterned, hypercellular atypia. What is his diagnosis and treatment?

A

Fibrosarcoma:
Malignant fibroblastic tumor
- Adults: seen in thigh, knee
- Congenital form= t(12;15)
- Firm mass

Histo: cellular fibroblastic proliferation with varying atypia

  • Pleomorphic, hypercellular
  • “herringbone pattern”

Treatment: surgical

Prognosis: 40% 5-year, directly related to differentiation

74
Q

A 73 year old man comes to the physician with a history of hip pain. On x-ray there appears to be an irregular lytic lesion in his pelvic fascia. Biopsy reveals the following histologic specimen. What is his diangosis and what complications arise from this type of lesion?

A

Undifferentiated high grade pleomorphic sarcoma:
Malignant fibrous histiocytoma
- Most COMMON soft tissue tumor
- seen in older adults

Sites: deep fascia, skeletal muscle

Histo: pleomorphic, storioform

Prognosis: dependent on cytologic atypia

  • 50% recur
  • Lung metastases
75
Q
A

Lipoma:
Most common benign soft tissue mass
- Sites: subcutaneous tissue, upper half of body

Gross: yellow, soft, lobulated
Histo: mature adipocytes

Treatment: simple excision
* Deeper: harder to differentiated from liposarcoma

76
Q

A 55 year old woman presents to her physician with a enlarging mass on her thigh. A biopsy revelas the following. What is her diagnosis and prognosis?

A

Liposarcoma
2nd most common sarcoma
- Adults, > 50 years
- Slow-growing, may be large (thigh, retroperitoneum)

Histologic subtypes:
- Myxoid variant: t(12;16)(q13,p11)

Prognosis: dependent on subtype (round cell type- < 20% 5 year survival)

77
Q

A 55 year old man presents to the doctor due to the development of a painful subcutaneous mass in his abdomen. A biopsy reveals the following. What is his diagnosis and treatment?

A

Leiomyoma:
Benign smooth muscle tumor
- Extrauterine: subcutaneous tissue
- May be painful

Histo: bland spindle cells, low mitotic rate

Treatment: simple excision

78
Q

A 2 year old girl is brought to the pediatrician because her father noticed a mass bulging from her vagina while changing her diaper. A sample is taken from the mass that reveals the following. What is her diagnosis and prognosis?

A

Rhabdomyosarcoma
Striated muscle differentiation
- Most common soft tissue sarcoma in children

HIsto: 5 types: prognosis best–> worst

  1. Botryoid embryonal: best prognosis
  2. Spindle
  3. Embryonal
  4. Alveolar
  5. Undifferentiated

Treatment: combined therapy

Prognosis: 80% 5-year

79
Q

A 20-year old woman comes to the doctor with increasing pain in her right knee. An x-ray reveals a mass in the joint space which the physician biopsies. The following is revealed. What is her diagnosis and prognosis?

A

Synovial sarcoma:
Arises in joint; highly malignant
- 10% soft tissue sarcomas
- Seen in adolescents, young adults
- Painful, tender mass

Histo: bi- or mono-phasic epithelial and spindle cells

60% recur; lung metastases

Prognosis: 50% at 5 years

80
Q

A 5 year old is brought to the emergency room for a suspected broken arm after falling off his bike in the driveway. This is his second fracture this year and the phsyician is concerned for abuse, but then changes his diagnosis when he performs an x-ray that reveals the following. What would a bone biopsy reveal in this diagnosis and what form does the child likely have? How should the child be treated?

A

Osteogenesis Imperfecta:
AD mutation in Type 1 collagen gene (Chr 17,7)–> delayed maturation of bone, joints, ears, ligaments, teeth, skin (7 types)

OI1= mildest form, blue sclerae

  • Fractures after birth
  • Exuberant callus formation (tumor-like)
  • Ossicles fuse–> deafness (age 20-30)
  • Normal stature

OI2= lethal prenatal form

  • Marked short stature, limb deformities
  • Crushed in utero

OI3= progressive, most severe type

  • multiple fractures present at brith
  • AD (rare AR)
  • Severely short stature
  • *OI4= similar to OI1 with normal sclera
  • Cortical bone woven–> eventually matures
  • Aggressively treated to prevent dwarfism**

Pathogenesis:
Mutations of Col1A1 (all OI), Col1A2 (OI 2, 3, 4) on chrom 17, 7
- Point mutation of glycine residue most common in OI
- Col1A2 mutation seen in 1/2 synthesized collagen molecules- variable phenotypes

81
Q

A 10 year old child is admitted to the hospital after he was brought to the emergency room screaming in pain. He fell and hurt his hip a few days ago and now he is running a fever and there is erythema around his flank on the side of injury. An x-ray reveals the following. What is the primary concern here?

A

Osteonecrosis: Legg-Calve-Perthese
Bone, marrow death; more common in children

Multiple causes:

  • Trauma
  • Emboli
  • Systemic disease
  • Steroids
  • Bisphosphonates

Necrotic bone: stress fracture and compaction over time–> fracture of rigid articular cartilage and subchondral bone

  • *Legg-Calve-Perthes:** Avascular necrosis of femoral head in children
  • Necrosis and remodeling–> collapse of subchondral bone, incongruous articular surface
  • May cause severe arthritis

Femoral head in kids depends in lateral epiphyseal vessels susceptible to trauma, intrarticular pressure

82
Q

A 12 year old girl with a recent salmonella infection is admitted to the hospital for a fever that will not abate. Additionally she complains of pain in her hip. An x-ray reveals the following in her femur. What is her diagnosis, how did it start, and what are the complications that could ensue?

A

Osteomyelitis:
Infection of bone (usually bacterial: staph, strep, haemophilus, salmonella) and marrow
- Direct penetration (trauma) or hematogenous
- Can also cross cartilage, infect joint spaces

Hematogenous: kids 5-15 years- may see fever, localized pain, etc
- Can also be seen in drug addicts

X-ray to diagnose aggressive process

  • *Pathogenesis:**
  • Unique blood supply at metaphyseal area–> slowing of blood
  • Bacteria penetrate wall–> infection
  • Increased pressure–> compromised blood supply–> necrosis–> more bacteria
  • Access to cortex, periosteum, joint, skin (claoca)
  • Sequestrum= dead cortical bone
  • Involucrum= new bone formation around dead bone
  • Brodie abscess= abscess

In vertebral bodes: disc are NOT barrier for spread

  • Enterics, staph–> attract inflammatory cells–> collagenase–> move to other cells
  • Complications: vertebral body collapse, spinal epidural abscess, cord compression
  • Vertebral body collapse, neurologic defects
  • *Complications:**
  • Septicemia
  • Acute bacterial arthritis (joint space)
  • Pathologic fracture
  • Squamous cell carcinoma (chronic infection)
  • Amyloidosis (chronic infection)
  • Chronic osteomyelitis
83
Q

A 55 year old man with a long-standing history of pain in his femur that does not change with activity finally sees his physician to relieve the pain. A bone marrow biopsy reveals the following. What is his diagnosis and treatment?

A

Chronic osteomyelitis:
Changes from acute inflammatory process–> chronic
- See plasma cells, lymphocytes
- Due to failure to resolve acute osteomyelitis

** Differentiate from myeloma, cytoma (plasma cell)- run immunohistochemical stains for light chains (polyclonal vs monoclonal)

Marrow fibrosis–> fibrosseous lesion

Treatment:
- Surgical debridement; long term antibiotics

84
Q

A 55 year old woman recently emigrated from India presents to the emergency room with radiculopathy in her lower limbs and uncontrolled urination. Based on the x-ray of her vertebral bodies (below), what has occured and what might have caused it?

A

TB bone infiltration:
From focus elsewhere
- Affects vertebral bodies; spares lamina, spines (Pott’s disease)
- Thoracic vertebrae (t-11> cervical or lumbar)
- Leads to collapse of vertebral body
- Can track down anterior soft tissue, spinal ligaments–> moves into pelvis, inguinal region (Cold abscess= psoas abscess)

Path:
Caseating granulomatous inflammation
- Slow resorption of bone
- Little or no woven bone–> collapse
- Special stains (AFB- organisms visible), look for sarcoid, fungi

85
Q

A 60 year old woman comes to the doctor due to a new onset of calcium stones in her urine as well as GI upset. Her physician runs blood labs and finds elevated PTH and calcium and the following x-ray shows changes in her bones. What is her diagnosis?

A

Hyperparathyroid bone disease:
Increased PTH–> bone resorption
- Early= line of osteoclasts–> flare lesions

  1. osteitis fibrosa (osteoclast stimulation–> marrow fibrosis)
  2. osteitis fibrosa cystica
    - -> brown tumor (hemosiderin deposition) : late stage
    - Subperiosteal bone resorption
86
Q

A 4 year old boy is brought to the pediatrician by his mother due to increasing incidents of falls. His mother is worried he is sick and he seems to be dropping his toys and breaking things more than usual. A muscle biopsy (below) reveals pathological changes. What might his diagnosis be and how is it confirmed?

A
  • *Duchenne Muscular Dystrophy**:
  • X-linked – most common non-inflammatory myopathy of children
  • Particularly affects pelvic and shoulder girdles
  • Women are carriers, however 30% of cases are spontaneous. (Dystrophin gene is HUGE)
  • Mutation of dystrophin localized to Xp21
  • Dystrophin links the cytoskeleton of the muscle cell to the sarcolemmal membrane.
  • DMD= complete or almost complete loss of dystrophin due to deletions of the gene.
  • *Clinical** DMD:
  • Weakness evident by age 3 or 4
  • Pseudohypertrophy of the calf muscles
  • Wheelchair bound by age 10 years
  • Bedridden by 15 years
  • Death due to respiratory insufficiency and/or cardiac arrhythmias
  • brain is also affected (mild mental retardation)
  • *Path**:
  • Dystrophin deficient fibers display osmotic fragility and undergo spontaneous necrosis.
  • necrosis of muscle fibers and regeneration
  • Regeneration is insufficient to keep pace and there is progressive loss of fibers and fibroadipose replacement.

Diagnosis: Immunohistochemical staining for dystrophin Western Blotting, PCR

* Becker’s muscular dystrophy: dystrophin protein present but it is usually truncated. As a result, the disease is milder.

Histo:
Early DMD:
- Small atrophic muscle fibers with enlarging fibers to compensate for atrophy
- Large fibers also have to work against fibrosis developing between muscle fibers

Late DMD:
- Significant fibrosis, more atrophic fibers, fatty infiltration

Dystrophin changes:
Normal: rim around muscle fibers= dystrophin
Absent dystrophin: need to due immunohistochemical stains to discover which proteins are lost

87
Q

A 65 year old woman comes to her physician complaining of increasing difficulty with performing household tasks. Additionally she has noticed her eyelids seem droopy and she has a harder time with some facial expressions- her husband told her she looks tired or bored all the time now. A muscle biopsy is taken and the following anomaly in sarcomere alignment is discovered. What is her diagnosis and what other manifestations might the physician find?

A

Myotonic dystrophy
Most common form of adult muscular dystrophy
- Characterized by Myotonia (difficulty in relaxation) and progressive muscle weakness

Two forms with autosomal dominant inheritance:

  1. DM1= CTG trinucleotide repeat syndrome
    - Repeats in the DM protein kinase (DMPK) gene (Serine-threonine protein kinase)
    - Normal - 30 copies of the CTG repeat
    - Affected > 50 copies
    - Congenital forms in children of affected DM1 mothers
    - slowly progressive muscle weakness and stiffness are seen, principally in the distal limbs
    * *- Facial and neck weakness as well as ptosis**
  2. DM2= CCTG repeats in the first intron of the ZNF9 gene
    - Zinc Finger Protein (binds RNA)
    - Proximal weakness more common than in DM1
  • *Extra skeletal muscular manifestations**:
  • Cataracts, testicular atrophy, personality deterioration.
  • Smooth muscle disorders of the GI tract and uterus.
  • Cardiac arrhythmias

** Congenital myotonic dystrophy is seen in children of affected mothers.

  • *Histo:** Sarcomeres are supposed to be parallel to axis–> shortening
  • Here they’re perpendicular–> squeeze (vs shorten)
88
Q

A 15 year old girl is having surgery following a car accident when she suddenly develops an increased core body temperature that the anaesthesiologist has to control by administering emergency dantrolene sodium and placing her in an ice bath. A muscle biopsy is taken after she survives this frightening episode and the folowing is found with NADPH staining. What is her diagnosis?

A

Central core disease
Type of congenital myopathy:
Autosomal Dominant

Ryanodine receptor mutation
- Calcium-release channel of sarcoplasmic reticulum.
* May be associated with malignant hyperthermia.

Muscle biopsy:

  • Type I fiber predominance with a central pale zone on oxidative stains (NADPH-Tr)
  • Ultrastructure shows loss of membranous organelles in the center of the fiber

Histo: NADH-stain= central pale zone (“central core disease)

89
Q

A 17 year old boy develops pain and weakness in his hips and shoulders over a period of weeks when he is finally brought to the doctor. A muscle biopsy reveals the following. What type of mutation is present and what is the disease called?

A
  • *Central nuclear myopathy**:
  • Heterogeneous group characterized by the presence of centrally placed nuclei.
    1. Recessive form
    2. Autosomal dominant, adolescent onset
  • Clinically resembles limb-girdle muscular dystrophy forms

Fibers resemble the myotubular stage in the embryogenesis of skeletal muscle
- Dynamin 2 - gene involved in membrane trafficking, centrosome and actin assembly.

90
Q

A 67 year old man presents with increasing muscle weakness concurrent to difficulty breathing. There are no vascular abnormalities noted (for a 67 year old man) so a muscle biopsy and a lung biopsy are performed. The muscle biopsy reveals the following- rimmed vacuoles with deposition of amyloid. What is his diagnosis and treatment?

A

Inclusion body myositis:
Inflammatory features similar to polymyositis, but seen in 60+ year old patients

Path:

  • Rimmed vacuoles (best seen on modified Gomori Trichrome Stain)- front
  • Increase in ragged red fibers
  • Congo Red positive amyloid (Texas red optics) material including: Beta Amyloid, phosphorylated tau, alpha synuclein, ubiquitin, presenilins, parkin, etc.
  • Intravacuolar and intranuclear filamentous inclusions (EM)
  • *Treatment:
  • IVIG**
  • _Non responsive to immunosuppression (l_ymphocytes respond to muscle damage- don’t cause it!)
91
Q

A 40 year old woman comes into your office complaining of increasing muscle weakness. Her face is elongated and on neuromuscular testing you notice bilateral facial weakness. A muscle biopsy is performed showing deposition of rod-like structures. What is her diagnosis?

A

Nemaline myopathy:
Heterogeneous group of muscle diseases characterized by the formation of rod like structures
- Autosomal dominant and recessive forms

Path:
Nemaline structures are electron dense and resemble Z-band material

Facies: elongated face, bifacial weakness

92
Q

A 3 week old baby is brought back to the pediatrician- the mother is distraught as her baby doesn’t seem to move as much as he did when he was born. On exam the pediatrician notes an enlarged tongue and abnormalities in the cardiac exam. A muscle biopsy is performed. What does the physician suspect and what is the prognosis?

A

Type II Glycogenosis:
AKA: Acid Maltase Deficiency, alpha 1,4 Glucosidase Deficiency, Pompe Disease
- Various mutations

Pathogenesis:

  • Acid Maltase is a lysosomal enzyme responsible for glycogen breakdown.
  • Deficiency results in lysosomal storage of glycogen (membrane bound)

Clinical:

  • More severe the earlier the age of onset
  • In the neonatal form, there is macroglossia, cardiac, liver, and skeletal muscle storage. CNS also affected. **Severe hypotonia
  • Later onset and adult forms, clinical expression limited to muscle, progressive muscle weakness.
  • *Histo**:
  • PAS staining demonstrates vacuolated fibers; glycogen leaches out–> empty space
  • Below: EM shows membrane-bound glycogen (dots)
93
Q

A 25 year old man comes to his physician due to increased muscle cramps over the past few months while exercising. A urine test reveals slight myoglobinuria following exercise earlier in the day. The physician performs a muscle biopsy and finds the following. What is his diagnosis, the pathology causing this condition and treatment?

A

Type V Glycogenosis
AKA: McArdle Disease, Myophosphorylase Deficiency
- Usually not progressive or debilitating

Myophosphorylase is skeletal muscle specific
- Myophosphoylase deficiency results in an inability to cleave glycogen and release glucose for energy utilization during physical exertion

Clinical:

  • _Muscle cramping
  • Avoid strenuous exercise otherwise muscle necrosis, myoglobinuria and renal failure_

Diagnosis:

  • Absence of myophosphorylase,
  • PAS positive storage of non-membrane bound glycogen

Below: subsarcolemmal glycogen deposition

94
Q

An 8 year old boy is brought to the family physician due to his decreased interest in sports and increasing clumsiness. His father noted he doesn’t like to play in soccer games any more and he seems to be losing weight. On physical exam the physician notes a loss of muscle tone with increased fat in his limbs. A muscle biopsy is performed that reveals the following increased adipose tissue in his muscle. What is his possible diagnosis, the underlying pathology and treatment?

A

Lipid storage myopathies: Carnitine deficiency
Clinical: Exercise intolerance

Path: Excess accumulation of neutral lipids (not CPT)
- Oil Red Orcein stains lipid red

Carnitine deficiency:
Carnitine synthesized in the liver
- Required for transport of Light Chain Fatty Acids into mitochondria
- Autosomal recessive

_Clinical:

  • Progressive muscle weakness and atrophy_
  • Signs of denervation and peripheral neuropathy
  • Massive accumulation of lipid in muscle

Treatment:
Oral carnitine may help

Other types of lipid storage myopathy:
Carnitine Palmitoyl Transferase deficiency:
Path:
Inability to metabolize LCFAs due to inability to transport them into the mitochondria

Clinical:

  • Prolonged exercise results in muscular pain, myoglobinuria
  • Prolonged fasting can do the same

Histo: No light microscopic changes

95
Q

A 5 year old girl is admitted to the hospital for a seizure. Her blood panels reveal lactic acidosis. Her mother states she wasn’t doing anything in particular when this started by she had noticed her daughter’s affect seemed to have changed a bit over the past few weeks. Based on the muscle biopsy below, what is her diagnosis and what is the underlying pathology?

A

Mitochondrial diseases: MELAS
Maternal inheritance for mitochondrial gene encoded proteins and tRNAs.
- Nuclear-encoded mitochondrial proteins are autosomally inherited
- Disease expression related to proportion of mutant mitochondria

Energy hungry organs especially CNS and muscle (cardiac and skeletal) are affected most:
- CNS= death of neurons

Path:

  • Accumulation of mitochondria (SDH)
  • Ragged Red fibers (also seen in inclusion body myositis)
  • Lipid and glycogen accumulation due to improper metabolism
  • Atrophy of muscle fibers
  • COX deficiency
  • *Histo:** Fiber with red around edge= increased number of mitochondria collecting in sub-sarcolemmal areas

* Ultrastructural abnormalities of mitochondria may be seen but are not present in all cases.

  • *Types of mitochondrial diseases**:
    1. Kearnes-Sayre syndrome (KSS):
  • progressive ophthalmoplegia, retinal pigmentary degeneration, cardiac arrhythmias
  • large deletions of mtDNA
  1. Chronic Progressive External Opthalmoplegia
    - Similar to KSS (only really seen in eyes): genetically autosomal dominant and recessive inheritance via nDNA
    - polymerase gamma (deletions), helicase, ANT1 (adenine nucleotide translocator-1)

3. MELAS-mitochondrial myopathy, encephalopathy, lactic acidosis and stroke-like episodes
- mutation in leucine tRNA mitochondrial-encoded gene

  1. MERRF-myoclonic epilepsy and ragged red fibers.
    - mutation in the lysine tRNA mitochondrial-encoded gene
96
Q

A 45 year old woman newly emigrated from South America is admitted to the hospital for increasing delirium and dementia-like symptoms. A physical exam reveals periorbital edema and cardiac abnormalities. The woman later expires from her symptoms and an autopsy is performed that reveals the following. What is her diagnosis?

A

Trichinosis:
Trichinella spiralis:
- Acquired from ingestion of undercooked meat from infected animals (mostly pork)
- Encysted larva develop into adult worms within intestine.
- Female worms release larvae that penetrate gut and encyst in skeletal muscle

Clinical:

  • often asymptomatic
  • pain, tenderness, eosinphilia, periorbital edema
  • infection of brain or myocardium may be fatal
97
Q

A 24 year old man is in a ski accident where he is buried in an avalanche. He is rescued but soon develops symptoms of renal failure. A biopsy of necrotic tissue removed from a leg that was crushed reveals the following. What is his diangosis?

A

Rhabdomyolysis:
Risk Factors:
- Secondary to metabolic myopathies
- Crush injuries
- May complicate influenza
- Heat stroke
- Malignant hyperthermia
- Alcoholism
- Sepsis

Other causes:

  • Noninflammatory destruction of fibers with phagocytosis
  • Myoglobinuria and renal failure
  • HMG CoA reductase inhibitors (Cholesterol lowering drugs such as Lipitor).

Labs: elevated creatinine kinase, kidney damage

98
Q

A patient on long-term high-dose steroids begins develop weakness in all four limbs and goes to see her doctor with concerns that she is developing MS. A muscle biopsy is taken that reveals the following. What is her diagnosis and how can she be treated?

A

Critical illness myopathy:
Also known as Acute Quadriplegic Myopathy, or Myosin Heavy Chain Depletion Syndrome:
- Classically results from the use of high-dose steroids and neuromuscular blocking agents.
- Profound loss of myosin (thick filaments) with preservation of other sarcomeric structure.
- Discontinuation of corticosteroids often results in reappearance of myosin filaments and recovery
- Cause unknown

Histo:
Normal Z-bands: dark lines= myosin
Critical illness: myosin gone (below)

99
Q

A 15 year old boy comes to the physician for an annual physical. On exam he has only gained 10 pounds in the last year, though he has grown 4 inches. He says he wants to build more muscle but hasn’t been able to get any additional muscle in spite of his strength training. A biopsy of his muscle would reveal the following. What is his diagnosis?

A

Spinal muscle atrophy
Degeneration of anterior horn cells
- Second most common, lethal, autosomal recessive disorder after cystic fibrosis

  1. Type I (Werdnig-Hoffman Disease, Infantile Spinal Muscular Atrophy):
    - Progressive and severe weakness in early infancy, death within 1-2 years.
    - Histology: atrophy of type I and II fibers with clusters of surviving hypertrophied type I fibers.
  2. Type II Intermediate

3. Type III Kugelberg-Welander Disease, Juvenile Spinal Muscular Atrophy:
- Later onset, not necessarily progressive
- Fiber type grouping
- Degenerating and regenerating fibers

Histo:
Grouped atrophy: some muscle fibers more intact, but eventually will atrophy with nervous system damage

100
Q

A 67 year old Caucasian woman comes to her physician with a persistant headache and pain of her scalp. Additionally she has muscle aches in her shoulders and hips. A vascular biopsy reveals the following. What is her diagnosis and treatment?

A

Giant cell arteritis:
Large Vessel Vasculitis

Used to be called temporal arteritis – but it can involve more than just the temporal artery

  • Primarily affects the cranial branches of the arteries originating from the aortic arch
  • May affect the aorta
  • Most common primary form of vasculitis
  • Seen primarily in Caucasians of N. European descent (not in AA), F > M, increases with age
  • *Clinical presentation:**
  • Severe unilateral headache with scalp tenderness
  • Jaw claudication and constitutional symptoms (fatigue, anorexia, weight loss)
  • Impaired blood flow to affected area causes symptoms: Vertebral (stroke), retinal (blindness)
  • Vision loss is most feared symptom, usually irreversible (anterior ischemic optic neuritis or retinal artery occlusion)

Polymyalgia rheumatica:
Patients report pain, stiffness in muscles of neck, shoulder, pelvic girdle
- Associated with signs of systemic inflammation such as elevated ESR, CRP
** Dramatically Responsive to low dose prednisone (vs arthritis, which is mildly responsive)

40-60% patients with GCA have PMR symptoms
- 15% of PMR patients will develop GCA; can be first symptom of GCA
** Low dose prednisone will not control GCA

Evaluation:

  • Physical exam: normal or show decreased pulse, thickened nodular artery
  • NO LAB testing: high ESR may be found
  • Biopsy can be helpful, not sensitive: biopsy every 1-2 cm to rule out “skip lesions”; biopsy both sides
  • No radiologic technique as specific as biopsy
  • *Treatment:**
  • High dose steroids given severity of manifestations such as blindness
  • *Path**:
  • Involved segments of the affected arteries develop nodular thickenings with reduction of the lumen and possible thrombosis.
  • Granulomatous inflammation of the media and intima centered on the internal elastic lamina. The internal elastica becomes irregular, fragmented and in advanced lesions can completely disappear.
  • Less commonly, granulomas and giant cells are absent, and there is a nonspecific panarteritis with mixed inflammatory infiltrate composed mainly of lymphocytes and macrophages with admixed eosinophils and neutrophils.
  • In late stages, intima is markedly thickened and the media is fibrotic.
101
Q

A 32 year old Asain woman presents to your office with dizziness, visual disturbances. Physical exam reveals absence of right radial pulse. Below is her angiogram. What is her diagnosis and treatment?

A

Takayasu’s arteritis
Large vessel vasculitis

VERY uncommon
Primarily affects young Asian women < 40 years old
- Affects large vessels: aorta, coronary, pulmonary arterties
“Pulseless disease”

Systemic signs and neurologic /cardiac symptoms
- May have claudication of extremities

Diagnosis: made with imaging but physical exam may show bruit
- Imaging of vessels: any evidence of claudication

Treatment: Steroids

  • *Path**:
  • Irregular thickening of the vessel wall with intimal hyperplasia; when the aortic arch is involved the lumens can be markedly narrowed or obliterated. Such narrowing explains the weakness of the peripheral pulses.
  • Histology: adventitial mononuclear infiltrates with perivascular cuffing of the vasa vasorum, intense mononuclear inflammation in the media, granulomatous inflammation, replete with giant cells and patchy medial necrosis.
  • The histologic appearance is indistinguishable from that of giant cell arteritis: based on age of patient
  • As the disease progresses, collagenous scarring, with admixed chronic inflammatory infiltrates, occurs in all three layers of the vessel wall. Occasionally, aortic root involvement causes aortic insufficiency.
102
Q

This is a arteriogram from a 45-year old man with fever, skin lesions and renal dysfunction. Additionally he has developed a foot drop. What is his most likely diagnosis and treatment?

A

Polyarteritis nodosum
Slight male predominence, broad age range
- Constitutional symptoms
- Strongly associated with Hep B infection

Symptoms depend on organ affected by commonly affects GI tract: may also see:

  • HTN, neuropathy, testicular pain, cutaneous disease
  • Spares lung and glomeruli (can involve renal artery with aneurysms)

Mononeuritis multiplex= Blood vessels that supply nerves attacked–> nerve death–> wrist/foot drop (most common cause of foot/wrist drop is diabetes)

Evaluation:

  • No specific diagnostic tests
  • May see elevated inflammatory markers
  • Mesenteric/renal angiography- aneurysms, irregular constrictions
  • NOT ANCA-associated vasculitis

Treatment:
Immunosuppression required due to 2-5 year mortality
- Degree of treatment guided by disease severity

  • *Pathology:**
  • Segmental transmural necrotizing inflammation of small and medium-sized arteries in any organ (kidney, heart, liver, GI) except the lung.
  • Individual lesions have a predilection for branching points, and may affect only part of the vessel circumference.
  • Characteristic for PAN: ALL STAGES of activity may coexist in different vessels or even within the same vessel, suggesting ongoing and recurrent insult.
  • Acute phase: Fibrinoid necrosis with significant acute inflammation surrounding the area of necrosis (with neutrophils, eosinophils, mononuclear cells).
  • Thrombosis of the lumen with secondary infarcts can occur.
  • As a result of injury to larger arteries small aneurysms can occur. Rupture of aneurysms located in critical areas can lead to a fatal hemorrhage
  • Late phase: Fibrous thickening of the vessel wall.
103
Q

A 2-year old boy is brought to his physician due to a fever, congestion, and rashes on his palms, soles and chest. Additionally there is blistering of the skin on his lips. What is his diagnosis, pathology, and treatment?

A

Kawasaki Disease:
Acute, self-limiting illness affecting young children, infants (85% < 5 years)
- More common in Japan

Symptoms:
- Fever, congestions, conjunctival infection, oral cavity dryness/redness with blistering, lymph node swelling, rash on palms and soles

May have later complications from cardiac disease

Path:
Pathology: P_AN-like vasculitis with necrosis and inflammation affecting the entire thickness of the vessel wall but with less prominent fibrinoid necrosis._ Pathologic changes outside the cardiovascular system are rarely significant!

Treatment: with intravenous immunoglobulin (IVIG) early (less than 10 days) decreases aneurysm formation by up to five-fold

104
Q

Below is an immunohistochemical stain of a blood vessel in a patient with chronic sinusitis, epistaxis and infections. What structure is stained below?

A

ANCA- associated vasculitis:
ANCA= anti-neutrophil cytoplasmic antibody
- C-ANCA (front): cytoplasmic antigens (PR3)= ELISA
- P-ANCA (below): perinuclear antigens (MPO, many other antigens- less specific)

3 types; overlap in clinical features:

  1. Wegener’s granulomatosis= granulomatosis with polyangiitis (GPA)
    - C-ANCA or PR3 in 80-90% patients
  2. Microscopic polyangiitis (MPA)
    - P-ANCA or MPO in 70% patients
    - Like GPA with no granulomas
  3. Churg-Strauss= eosinophillic granulomatosis with polyangiitis (EGPA)
    - P-ANCA or MPO in ~50% patients

Diseases treated similarly: immuno-suppressants chosen based on degree of organ involvement

105
Q

A 45 year old woman comes to your office after suffering from chonic sinus infections and now some increasing nosebleeds. On physical exam you note that her face seems to have changed since the last time you saw her. A blood draw reveals elevated C-ANCA titers. What is her diagnosis and treatment?

A

Granulomatosis with Polyangitis
M=F, peak in middle age
- Patients usually have constitutional symptoms
- Limited form: only involves upper respiratory tract and/or lower respiratory tract
- Upper respiratory symptoms: chronic sinusitis, epistaxis, “infections”
- Can see nasal perforations, “saddle nose” deformity

Lower respiriatory symptoms: range from pulmonary infiltrates to alveolar hemorrhage

  • Cavitary nodules
  • Glomerulonephritis

Likely to see increased inflammatory markers: ESR, CRP, ANCA
- C-ANCA or PR3 in 80-90% patients

Treatment: immunosupression

106
Q

A 43 year old man comes to the physician after trying for months to control his asthma unsuccessfully. He has had 3 visits to the ER in the last two months for asthma attacks. A physician in the ER suspects his symptoms might not be strictly asthma. A vascular biopsy is taken which reveals the following. What is his diagnosis and what other tests should be ordered to confirm this?

A

Eosinophilic granulomatosis with polyangiitis
M=F, mean age= 40
- Majority of patients have asthma, atopy, allergy (ex: asthma that patient can’t control with meds)
- Can affect MULTIPLE organ systems (not just lung)
- Symptoms depend on affected organ, can also include recalcitrant asthma

Found with eosinophilia:

  • Biopsy shows eosinophilic inflammation, necrotizing vasculitis
  • *- Associated with P-ANCA, MPO**
107
Q

A 57 year old woman comes to her physician with a new rash on her leg. A few years ago she was diagnosed with Raynaud’s phenomenon and her physician suspects this may be another manifestation of her syndrome. What type of blood tests should she order?

A

Cryoglobulinemic vasculitis:
Ig precipitate in cold (keep samples warm for diagnosis)
1. Type 1= monclonal antibody (IgG)
2. Type 2= Polyclonal IgG and monoclonal IgM RF
- Often due to Hep C (treat Hep C, cure vasculitis), HIV
3. Type 3= polyclonal immunoglobulins

Thought to induce disease through immune complex activation, deposition in vessels

Clinical features:

  • Related to organ involvement
  • May see digital ulceration/cyanosis, Raynaud’s phenomenon
  • Neuropathy, renal failure, rash
108
Q

A 6 year old boy is brought to the pediatrician due to new rash on his legs. His mother says a week ago he had a cold and then this rash appeared. Additionally he says his knees are hurting. What is his diagnosis and treatment?

A

IgA vasculitis
AKA Henoch-Schonlein purpura
- Primarily affects children ~ 5 years
- Most commonly diagnosed vasculitis in children
- Common Upper respiratory tract infection followed by purpuric rash, arthralgias, abdominal pain
- May also have glomerulonephritis

Treatment: supportive (self-limiting in children)

109
Q

A 37 year old woman comes into her physician complaining of an itchy rash that has been bothering her for the past two days. Upon exam it is apparent that she has a urticarial rash, but it is unclear why it has lasted for more than a few hours. Additionally the physician notes that the patient has been using eye drops and complains that her eyes feel dry and scratchy. What labs should the physician order and what might the diagnosis be?

A

Hypocomplementeric urticaric vasculitis
Patients have urticarial lesions that lasts > 24 hours and SCAR
- May also have arthritis, arthralgias, glomerulonephritis, uveitis, episcleritis, pulmonary involvement, abdominal pain

Labs:
- Low complements with l_ow C1q level, anti-C1q antibodies_

May be from primary autoimmune disease- associated with other etiologies

110
Q

A 65 year old man of Israeli descent comes to his physician for the appearance of large, painful ulcers in his mouth. Additionally he has an ulceration on his genitals and is concerned he contracted an STD but hasn’t had any sexual partners in several years. What is his possible diagnosis?

A
  • *Variable vessel vasculitis**
  • *Behcet’s disease**= Severe aphthous ulcers, genital ulcers, skin lesions, ocular involvement, all size blood vessels
  • More common in Middle Eastern male patients

Pathergy: pustule forms where IV inserted into arm (unique to Behcet’s)

Cogan’s syndrome= Inflammatory disorder affecting eyes and ears, no direct test

111
Q

A 55 year old man comes to the emergency room after a rash sprouts up on his feet that causes him a lot of concern. The physician notes that the marks are palpable. The patient says he was recently started on a new medication and he wonders if this could have caused this problem. What is his possible diagnosis and treatment?

A

Cutaneous leukocytoclastic angiitis:
Characterized by immune complex deposition in arterioles, venules, capillaries

Systemic or primarily cutaneous: palpable purpura, maculopapular rash

Biopsy: cutaneous leukocytoclastic angiitis- not confirmatory for autoimmune disease

Therapy: remove inciting agent, immuno-suppression